Diễn Đàn MathScopeDiễn Đàn MathScope
  Diễn Đàn MathScope
Ghi Danh Hỏi/Ðáp Community Lịch

Go Back   Diễn Đàn MathScope > Sơ Cấp > Đại Số và Lượng Giác

News & Announcements

Ngoài một số quy định đã được nêu trong phần Quy định của Ghi Danh , mọi người tranh thủ bỏ ra 5 phút để đọc thêm một số Quy định sau để khỏi bị treo nick ở MathScope nhé !

* Nội quy MathScope.Org

* Một số quy định chung !

* Quy định về việc viết bài trong diễn đàn MathScope

* Nếu bạn muốn gia nhập đội ngũ BQT thì vui lòng tham gia tại đây

* Những câu hỏi thường gặp

* Về việc viết bài trong Box Đại học và Sau đại học


Trả lời Gởi Ðề Tài Mới
 
Ðiều Chỉnh Xếp Bài
Old 29-07-2011, 06:39 PM   #16
batigoal
Super Moderator
 
batigoal's Avatar
 
Tham gia ngày: Jul 2010
Đến từ: Hà Nội
Bài gởi: 2,895
Thanks: 382
Thanked 2,968 Times in 1,295 Posts
Trích:
Nguyên văn bởi Kratos View Post
[FONT="Book Antiqua"]Mình post lại một số bài toán ở topic bất đẳng thức mà không có lời giải. Hi vọng mọi người hỗ trợ đóng góp lời giải cho tuyển tập.

Bài 2. Cho $a,b,c $ là các số thực dương thoả mãn $a+b+c=3 $. Chứng minh rằng
[*]
$\dfrac{1}{\sqrt{2a^2+ab+bc}} + \dfrac{1}{\sqrt{2b^2+bc+ca}} + \dfrac{1}{\sqrt{2c^2+ca+ab}}\ge \dfrac{3}{2} $
Chúng tôi có:
$2a^2+ab+bc=a^2+a(a+b+c)=a^2+3a $.
BĐT đã cho đưa về:$\frac{1}{\sqrt{a^2+3a}}+\frac{1}{\sqrt{b^2+3b}}+\f rac{1}{\sqrt{c^2+3c}}\geq \frac{3}{2} $.
Xét hàm số $f(t)=\frac{1}{\sqrt{t^2+3t}} $ là xong.
[RIGHT][I][B]Nguồn: MathScope.ORG[/B][/I][/RIGHT]
 
__________________
“ Sức mạnh của tri thức là sự chia sẻ tri thức”

[Only registered and activated users can see links. ]
batigoal is offline   Trả Lời Với Trích Dẫn
The Following User Says Thank You to batigoal For This Useful Post:
n.v.thanh (29-07-2011)
Old 29-07-2011, 06:42 PM   #17
Kratos
+Thành Viên+
 
Tham gia ngày: Nov 2009
Đến từ: Toán 0912, PTNK, Tp.HCM
Bài gởi: 87
Thanks: 25
Thanked 160 Times in 73 Posts
Gửi tin nhắn qua Yahoo chát tới Kratos
Trích:
Nguyên văn bởi batigoal View Post
Chúng tôi có:
$2a^2+ab+bc=a^2+a(a+b+c)=a^2+3a $.
BĐT đã cho đưa về:$\frac{1}{\sqrt{a^2+3a}}+\frac{1}{\sqrt{b^2+3b}}+\f rac{1}{\sqrt{c^2+3c}}\geq \frac{3}{2} $.
Xét hàm số $f(t)=\frac{1}{\sqrt{t^2+3t}} $ là xong.
$2a^2+ab+bc $ đâu có bằng $a^2+a(a+b+c) $ ạ :-s

Anh batigoal xem kĩ dùm em ạ, em không có ý gì nhưng anh biến đổi hay có chút nhầm lẫn :-s

[RIGHT][I][B]Nguồn: MathScope.ORG[/B][/I][/RIGHT]
 
Kratos is offline   Trả Lời Với Trích Dẫn
The Following User Says Thank You to Kratos For This Useful Post:
n.v.thanh (29-07-2011)
Old 29-07-2011, 06:43 PM   #18
batigoal
Super Moderator
 
batigoal's Avatar
 
Tham gia ngày: Jul 2010
Đến từ: Hà Nội
Bài gởi: 2,895
Thanks: 382
Thanked 2,968 Times in 1,295 Posts
Ừm, anh xin lỗi. Ngồi máy tính từ trưa đến giờ chắc mụ đầu rồi. nhầm lẫn hết.
[RIGHT][I][B]Nguồn: MathScope.ORG[/B][/I][/RIGHT]
 
__________________
“ Sức mạnh của tri thức là sự chia sẻ tri thức”

[Only registered and activated users can see links. ]
batigoal is offline   Trả Lời Với Trích Dẫn
The Following User Says Thank You to batigoal For This Useful Post:
n.v.thanh (29-07-2011)
Old 29-07-2011, 06:58 PM   #19
batigoal
Super Moderator
 
batigoal's Avatar
 
Tham gia ngày: Jul 2010
Đến từ: Hà Nội
Bài gởi: 2,895
Thanks: 382
Thanked 2,968 Times in 1,295 Posts
Trích:
Nguyên văn bởi ladykillah96 View Post
Cho em post nhờ mấy bài cũng chưa có lời giải trong Topic bất đẳng thức vào trong này luôn nhé!
c
Bài 12. Cho $a, b, c > 0 $. Chứng minh rằng:
$\sqrt{\frac{a^3 + bc}{b^2 + bc + c^2}} + \sqrt{\frac{b^3 + ca}{c^2 + ca + a^2}} + \sqrt{\frac{c^3 + ab}{a^2 + ab + b^2}} \ge \sqrt6 $
Bài này đề bài sai.
Với $a=b=c=\frac{1}{2} $ thì $VT=2,12<VP=2,4 $
Trong khi đề$VT\ge VP $
[RIGHT][I][B]Nguồn: MathScope.ORG[/B][/I][/RIGHT]
 
__________________
“ Sức mạnh của tri thức là sự chia sẻ tri thức”

[Only registered and activated users can see links. ]
batigoal is offline   Trả Lời Với Trích Dẫn
Old 29-07-2011, 07:35 PM   #20
chém gà
+Thành Viên+
 
Tham gia ngày: Jul 2011
Đến từ: France
Bài gởi: 136
Thanks: 8
Thanked 60 Times in 54 Posts
Bài 4 hình như sai đề hay sao ấy.Mình thấy có một bài toán với giả thiết y hệt và cũng tìm min nhưng đề bài là $\sum \frac{x}{x^2+1} $ và min rất khó bằng $-\frac{1}{2} $

Mình nghĩ là sai đề vì chưa có lời giải mà.
Đề bài như thế có lẽ cũng cm được không tồn tại min hay sao ấy.
Các bạn thử máy tính xem.
[RIGHT][I][B]Nguồn: MathScope.ORG[/B][/I][/RIGHT]
 
__________________
Gà nhất quả đất-Nát nhất Sever

chém gà is offline   Trả Lời Với Trích Dẫn
Old 30-07-2011, 12:39 AM   #21
daiduong1095
+Thành Viên+
 
daiduong1095's Avatar
 
Tham gia ngày: Sep 2010
Đến từ: CVP-Math
Bài gởi: 287
Thanks: 13
Thanked 210 Times in 112 Posts
Gửi tin nhắn qua Yahoo chát tới daiduong1095
Trích:
Nguyên văn bởi ladykillah96 View Post
Bài 10. Cho $a, b, c > 0 $. Chứng minh rằng:
$\left(8 + \dfrac{2\sqrt{ab}}{a + b}\right).\sqrt[3]{a.\dfrac{a + b}{2}.\dfrac{a + b + c}{3}} \ge 3(a + \sqrt{ab} + \sqrt[3]{abc}) $
Bài này có thể làm như sau:
Áp dụng BDT Horlder ta có:
$\sqrt[3]{\frac{a+a+a}{3}.\frac{a+\sqrt{ab}+b}{3}.\frac{a+b +c}{3}} \ge \frac{a+\sqrt{ab}+\sqrt[3]{abc}}{3} $
Do vậy ta chỉ cần cm được rằng:
$\left(8 + \dfrac{2\sqrt{ab}}{a + b}\right).\sqrt[3]{a.\dfrac{a + b}{2}.\dfrac{a + b + c}{3}} \ge 9\sqrt[3]{\frac{a+a+a}{3}.\frac{a+\sqrt{ab}+b}{3}.\frac{a+b +c}{3}} $
Hay $\left(8 + \dfrac{2\sqrt{ab}}{a + b}\right).\sqrt[3]{\frac{a+b}{2}} \ge 9\sqrt[3]{\frac{a+\sqrt{ab}+b}{3}} $
Mà theo BDT AM-GM thì:
$\left(8 + \dfrac{2\sqrt{ab}}{a + b}\right)\sqrt[3]{\frac{a+b}{2}} =\left(3+3+\frac{2(a+\sqrt{ab}+b)}{a+b} \right)\sqrt[3]{\frac{a+b}{2}} \ge 3\sqrt[3]{3^2.\frac{2(a+\sqrt{ab}+b)}{a+b}.\frac{a+b}{2}} $
$=9\sqrt[3]{\frac{a+\sqrt{ab}+b}{3}} $
Vậy bài toán cm xong.
[RIGHT][I][B]Nguồn: MathScope.ORG[/B][/I][/RIGHT]
 
__________________
daiduong1095 is offline   Trả Lời Với Trích Dẫn
The Following 2 Users Say Thank You to daiduong1095 For This Useful Post:
anh_96 (06-08-2011), ladykillah96 (30-07-2011)
Old 30-07-2011, 11:09 AM   #22
_minhhoang_
+Thành Viên+
 
Tham gia ngày: Sep 2010
Đến từ: Saint Petersburg
Bài gởi: 126
Thanks: 18
Thanked 221 Times in 75 Posts
Gửi tin nhắn qua Yahoo chát tới _minhhoang_
Trích:
Nguyên văn bởi Kratos View Post
Bài 2. Cho $a,b,c $ là các số thực dương thoả mãn $a+b+c=3 $. Chứng minh rằng[*]
$\dfrac{1}{5a^2+ab+bc}+\dfrac{1}{5b^2+bc+ca}+\dfrac {1}{5c^2+ca+ab} \ge \dfrac{3}{7} $
Áp dụng BDT Cauchy-Schwarz, được:
$\frac{1}{{5{a^2} + ab + bc}} + \frac{1}{{5{b^2} + bc + ca}} + \frac{1}{{5{c^2} + ca + ab}} $
$ = \sum\limits_{cyc} {\frac{{{{(b + c)}^2}}}{{{{(b + c)}^2}(5{a^2} + ab + bc)}}} $
$ \ge \frac{{4{{(a + b + c)}^2}}}{{\sum\limits_{cyc} {{{(b + c)}^2}(5{a^2} + ab + bc)} }} $
Ta cần chứng minh:
$\frac{{4{{(a + b + c)}^2}}}{{\sum\limits_{cyc} {{{(b + c)}^2}(5{a^2} + ab + bc)} }} \ge \frac{3}{7} $
$\Leftrightarrow 28{(a + b + c)^4} \ge 27(\sum\limits_{cyc} {{{(b + c)}^2}(5{a^2} + ab + bc)} ) $
$\Leftrightarrow 28\sum {{a^4}} + 58\sum\limits_{cyc} {{a^3}b} + 85\sum\limits_{cyc} {a{b^3}} \ge 156\sum {{a^2}{b^2}} + 15abc(a + b + c ) $
Theo BDT AM-GM, dễ thấy:
$58\sum\limits_{cyc} {{a^3}b} + 58\sum\limits_{cyc} {a{b^3}} \ge 116\sum {{a^2}{b^2}} $
$27\sum {{a^4}} + 27\sum\limits_{cyc} {a{b^3}} \ge 54\sum {{a^2}{b^2}} $
$\sum {{a^4}} + 14\sum {{a^2}{b^2}} \ge 15abc(a + b + c) $
Cộng lại ta có dpcm
[RIGHT][I][B]Nguồn: MathScope.ORG[/B][/I][/RIGHT]
 
__________________
Хоанг
_minhhoang_ is offline   Trả Lời Với Trích Dẫn
The Following 2 Users Say Thank You to _minhhoang_ For This Useful Post:
Kratos (30-07-2011), n.v.thanh (30-07-2011)
Old 30-07-2011, 12:36 PM   #23
_minhhoang_
+Thành Viên+
 
Tham gia ngày: Sep 2010
Đến từ: Saint Petersburg
Bài gởi: 126
Thanks: 18
Thanked 221 Times in 75 Posts
Gửi tin nhắn qua Yahoo chát tới _minhhoang_
Trích:
Nguyên văn bởi ladykillah96 View Post
Bài 12. Cho $a, b, c > 0 $. Chứng minh rằng:
$\sqrt{\frac{a^3 + bc}{b^2 + bc + c^2}} + \sqrt{\frac{b^3 + ca}{c^2 + ca + a^2}} + \sqrt{\frac{c^3 + ab}{a^2 + ab + b^2}} \ge \sqrt6 $
Bài này hình như sai đề rồi, biểu thức trên tử không đồng bậc
------------------------------
Trích:
Nguyên văn bởi Kratos View Post
$\sqrt{4a^2+(b-c)^2}+\sqrt{4b^2+(c-a)^2}+\sqrt{4c^2+(a-b)^2} \ge 3(\sqrt{a^2+abc}+\sqrt{b^2+abc}+\sqrt{c^2+abc}) $
Bất đẳng thức này cũng k đúng, thay với $a=b=c = \frac{1}{3} $
[RIGHT][I][B]Nguồn: MathScope.ORG[/B][/I][/RIGHT]
 
__________________
Хоанг

thay đổi nội dung bởi: _minhhoang_, 30-07-2011 lúc 01:00 PM Lý do: Tự động gộp bài
_minhhoang_ is offline   Trả Lời Với Trích Dẫn
The Following User Says Thank You to _minhhoang_ For This Useful Post:
n.v.thanh (30-07-2011)
Old 30-07-2011, 01:38 PM   #24
nguoivn
Banned
 
Tham gia ngày: Feb 2011
Đến từ: Hà Nội
Bài gởi: 42
Thanks: 2
Thanked 66 Times in 26 Posts
Trích:
Nguyên văn bởi batigoal View Post
Đành giở chiêu Càn long thập bát chưởng Óanh bài này theo phương pháp không thuần túy xem sao:
Xét hàm số :
$F(t)=\left(\frac{a}{4a+5b+3c}\right)^t+ \left(\frac{b}{4b+5c+3a}\right)^t+ \left(\frac{c}{4c+5a+3b}\right)^t $ với $t>0 $.
Chúng ta sẽ chứng minh hàm $F(t) $ đồng biến trên$(0;+\infty) $.
Thật vậy với $t_1<t_2 $ chúng ta sẽ chứng minh $F(t_1)<F(t_2) $.
Đặt $T=\left(\frac{a}{4a+5b+3c}\right)^{t_1}+ \left(\frac{b}{4b+5c+3a}\right)^{t_1}+ \left(\frac{c}{4c+5a+3b}\right)^{t_1} $
Áp dụng BDT becnuli, chúng tôi có:
$\left(\frac{a}{4a+5b+3c}\right)^{t_2}+\frac{t_2}{t _1}-1\geq \frac{t_2}{t_1}\left(\frac{a}{4a+5b+3c}\right)^{t_ 1} $
Tương tự cho 2 Bdt còn lại.
Ngoài ra ta có:$(\frac{t_2}{t_1}-1)T\geq (\frac{t_2}{t_1}-1)\frac{3}{12^{\frac{2}{3}}} $.
Cộng theo vế 4 bdt trên ta được $F(t_2)\ge F(t_1) $.
Vầy F(t) là hàm đồng biến với $t>0 $ .
Từ đó với $ \alpha \leq \beta $ta có:
$\left(\frac{a}{4a+5b+3c}\right)^{\alpha}+ \left(\frac{b}{4b+5c+3a}\right)^{\alpha}+ \left(\frac{c}{4c+5a+3b}\right)^{\alpha} \le \left(\frac{a}{4a+5b+3c}\right)^{\beta }+ \left(\frac{b}{4b+5c+3a}\right)^{\beta }+ \left(\frac{c}{4c+5a+3b}\right)^{\beta } $
Dấu "=" khi $a=b=c $
Hình như là problem cũ của mình, nhưng đọc cái "chứng minh" này thấy buồn cười tí :p
Batigoal xem lại xem em đang chứng minh cái gì nhé
PS: Bài này có thể giải được bằng AM-GM kết hợp CS.
[RIGHT][I][B]Nguồn: MathScope.ORG[/B][/I][/RIGHT]
 
nguoivn is offline   Trả Lời Với Trích Dẫn
Old 30-07-2011, 02:56 PM   #25
batigoal
Super Moderator
 
batigoal's Avatar
 
Tham gia ngày: Jul 2010
Đến từ: Hà Nội
Bài gởi: 2,895
Thanks: 382
Thanked 2,968 Times in 1,295 Posts
Trích:
Nguyên văn bởi nguoivn View Post
Hình như là problem cũ của mình, nhưng đọc cái "chứng minh" này thấy buồn cười tí :p
Batigoal xem lại xem em đang chứng minh cái gì nhé
PS: Bài này có thể giải được bằng AM-GM kết hợp CS.
1.Bài này post nêu ý tưởng mà giờ mới thấy có ý kiến góp ý, cảm ơn bạn vì nhận xét.
Đúng là bài này cần xem xét kĩ với t>0 nên tạm thời gỡ bỏ.
Ở trên sử dụng bdt Becnuli nội suy với tam thức bậc $(\alpha,\
beta) $.
2.Batigoal cũng cũng không phải người đi sâu về bdt, nên việc sử dụng AM-GM,hay C-S kĩ năng chỉ dừng ở mức hiểu và biết cách vận dụng và phân tích lời giải còn để đạt đến độ thành thục, nhuần nhuyễn thì còn phải học hỏi nhiều ở mọi người, nhưng thích những ý tưởng mới nên cứ post lời giải kiểu mới lên, dù biết có thể chưa đúng nhưng cũng sẽ được nhận xét để học hỏi..
[RIGHT][I][B]Nguồn: MathScope.ORG[/B][/I][/RIGHT]
 
__________________
“ Sức mạnh của tri thức là sự chia sẻ tri thức”

[Only registered and activated users can see links. ]

thay đổi nội dung bởi: batigoal, 30-07-2011 lúc 03:08 PM
batigoal is offline   Trả Lời Với Trích Dẫn
Old 30-07-2011, 06:14 PM   #26
_minhhoang_
+Thành Viên+
 
Tham gia ngày: Sep 2010
Đến từ: Saint Petersburg
Bài gởi: 126
Thanks: 18
Thanked 221 Times in 75 Posts
Gửi tin nhắn qua Yahoo chát tới _minhhoang_
Trích:
Nguyên văn bởi daiduong1095 View Post
Nếu a,b,c không là 3 cạnh của một tam giác thì $VT \le0 <1 $
Xét khi a,b,c là 3 cạnh của một tam giác đặt $x=b+c-a,y=c+a-b,z=a+b-c $
Khi đó x,y,z là các số thực dương và ta cần phải cmr $xyz \le1 $.
Phản chứng giả sử tồn tại x,y,z dương mà $xyz>1 $
Giả thiết biến đổi thành $x+y+z =\frac{2}{x+y}+\frac{2}{y+z}+\frac{2}{z+x} $
$\Leftrightarrow (x+y+z)^2(xy+yz+zx)=2(x+y+z)^2+2(xy+yz+zx)+xyz(x+y +z) $(1)
Mà theo BDT AM-GM ta có:
$\frac{2}{3}(x+y+z)^2(xy+yz+zx)> 2(x+y+z)^2 $

$\frac{2}{9}(x+y+z)^2(xy+yz+zx)>2(xy+yz+zx) $

$\frac{1}{9}(x+y+z)^2(xy+yz+zx)>xyz(x+y+z) $
Do $xyz>1 $
Cộng vế theo vế ta thấy mâu thuẫn với (1) $\Rightarrow $ đpcm.
Còn một cách khác ,theo đề bài ta chứng minh:
$ {\left( {{\rm{(b + c - a)(c + a - b)(a + b - c)}}} \right)^2} \le {(\frac{{a + b + c}}{{\frac{1}{a} + \frac{1}{b} + \frac{1}{c}}})^3} $
$ \Leftrightarrow {\left( {{\rm{(b + c - a)(c + a - b)(a + b - c)}}} \right)^2} \le {\left( {\frac{{abc(a + b + c)}}{{ab + bc + ca}}} \right)^3} $
Với a,b,c k là 3 cạnh tam giác thì bdt đúng, với a,b,c là 3 cạnh tam giác, theo cách dặt như trên thì ta cần chứng minh:
$8{(xyz)^2}{({x^2} + {y^2} + {z^2} + 3(xy + yz + zx))^3} \le {(x + y)^3}{(y + z)^3}{(z + x)^3}{(x + y + z)^3 $
BDT này đúng vì
${({x^2} + {y^2} + {z^2} + 3(xy + yz + zx))^3} \le 8{(a + b + c)^6} $
$(x + y)(y + z)(z + x) \ge \frac{8}{9}(a + b + c)(ab + bc + ca) $
$ {(ab + bc + ca)^3} \ge 27{(xyz)^2} $
Đây đều là những bdt quen thuộc, ta có dpcm, dấu "=" xảy ra khi a=b=c
[RIGHT][I][B]Nguồn: MathScope.ORG[/B][/I][/RIGHT]
 
__________________
Хоанг
_minhhoang_ is offline   Trả Lời Với Trích Dẫn
The Following 2 Users Say Thank You to _minhhoang_ For This Useful Post:
Kratos (30-07-2011), MathForLife (30-07-2011)
Old 30-07-2011, 08:11 PM   #27
_minhhoang_
+Thành Viên+
 
Tham gia ngày: Sep 2010
Đến từ: Saint Petersburg
Bài gởi: 126
Thanks: 18
Thanked 221 Times in 75 Posts
Gửi tin nhắn qua Yahoo chát tới _minhhoang_
Trích:
Nguyên văn bởi Kratos View Post
Bài 4. Cho $x,y,z $ là các số thực thỏa mãn $x+y+z=xy+yz+zx $. Tìm giá trị nhỏ nhất của biểu thức

$P=\dfrac{xy}{1+x^2}+\dfrac{yz}{1+y^2}+\dfrac{zx}{1 +z^2} $
Bài này không tồn tại GTNN , với x,y thực, chọn $z = \frac{x+y-xy}{x+y-1} $. Chọn x, cố định, cho y --> - vô cực thì z --> -x, nên P --> - vô cực,
[RIGHT][I][B]Nguồn: MathScope.ORG[/B][/I][/RIGHT]
 
__________________
Хоанг
_minhhoang_ is offline   Trả Lời Với Trích Dẫn
The Following User Says Thank You to _minhhoang_ For This Useful Post:
Kratos (30-07-2011)
Old 30-07-2011, 11:20 PM   #28
_minhhoang_
+Thành Viên+
 
Tham gia ngày: Sep 2010
Đến từ: Saint Petersburg
Bài gởi: 126
Thanks: 18
Thanked 221 Times in 75 Posts
Gửi tin nhắn qua Yahoo chát tới _minhhoang_
Trích:
Nguyên văn bởi ladykillah96 View Post

Bài 8. Cho $a, b, c $ là độ dài ba cạnh của tam giác. Chứng minh rằng:
  1. $\frac{a + b}{c^2}.(a + b - c) + \frac{b + c}{a^2}.(b + c - a) + \frac{c + a}{b^2}.(c + a - b) \ge \frac{18}{ab + bc + ca} $
  2. $\frac{1}{\sqrt{b + c - a}}+\frac{1}{\sqrt{c + a - b}} + \dfrac{1}{\sqrt{a + b - c}} \le \frac{1}{3}\left(\sqrt[4]{\frac{a}{bc}} + \sqrt[4]{\frac{b}{ca}} + \sqrt[4]{\frac{c}{ab}}\right)^2 $
Câu 1 2 vế không đồng bậc, bỏ cái mẫu $ab + bc + ca $ đi thì đúng
Câu 2 đề đúng phải là
$\frac{1}{\sqrt{b + c - a}}+\frac{1}{\sqrt{c + a - b}} + \dfrac{1}{\sqrt{a + b - c}} \ge \frac{1}{3}\left(\sqrt[4]{\frac{a}{bc}} + \sqrt[4]{\frac{b}{ca}} + \sqrt[4]{\frac{c}{ab}}\right)^2 $
[RIGHT][I][B]Nguồn: MathScope.ORG[/B][/I][/RIGHT]
 
__________________
Хоанг
_minhhoang_ is offline   Trả Lời Với Trích Dẫn
Old 31-07-2011, 10:13 PM   #29
Kratos
+Thành Viên+
 
Tham gia ngày: Nov 2009
Đến từ: Toán 0912, PTNK, Tp.HCM
Bài gởi: 87
Thanks: 25
Thanked 160 Times in 73 Posts
Gửi tin nhắn qua Yahoo chát tới Kratos
Rất cám ơn sự đóng góp của các bạn.

Nhân tiện đây, mình post thêm 1 bài nữa cũng chưa có lời giải. Hi vọng tiếp tục nhận được sự quan tâm của mọi người.

Bài toán. Cho $a,b,c $ là các số thực dương thay đổi bất kì. Chứng minh rằng

$\dfrac{b+c}{2a^2+bc}+\dfrac{c+a}{2b^2+ca}+\dfrac{a +b}{2c^2+ab} \ge \dfrac{6}{a+b+c} $

[RIGHT][I][B]Nguồn: MathScope.ORG[/B][/I][/RIGHT]
 
Kratos is offline   Trả Lời Với Trích Dẫn
The Following User Says Thank You to Kratos For This Useful Post:
n.v.thanh (12-08-2011)
Old 07-08-2011, 12:31 PM   #30
asdfghj
+Thành Viên+
 
asdfghj's Avatar
 
Tham gia ngày: Sep 2010
Đến từ: Hải Dương
Bài gởi: 214
Thanks: 139
Thanked 128 Times in 71 Posts
Tiếp mấy bài nữa nhé. Mọi người làm nhanh lên được thì tốt
1. Cho $a,b,c,d $ thỏa $ab=c^2+4d^2 $
Chứng minh $(a-c)^2+(b-d)^2\geq \dfrac{8}{5} $
2.Cho $a, b, c >0 $
Chứng minh $\sum \sqrt{\dfrac{8a^2+bc}{b^2+c^2}}\geq \dfrac{9}{\sqrt{2}} $
3. Cho $a, b, c ,d \geq 0, \sum a^2=1 $
Tìm min
$\dfrac{a}{1+bcd}+\dfrac{b}{1+acd}+\dfrac{c}{1+bad} +\dfrac{d}{1+bca} $
4. Cho a, b, c thỏa mãn $\dfrac{1}{2}\leq a;b;c\leq 2 $
chứng minh rằng
$\dfrac{a}{a+b}+\dfrac{b}{b+c}+\dfrac{c}{c+a} $
5. Tìm GTLN với $a, b, c>0 $
$\dfrac{1}{\sqrt{a^2+b^2+c^2}}-\dfrac{2}{(a+1)(b+1)(c+1)} $
[RIGHT][I][B]Nguồn: MathScope.ORG[/B][/I][/RIGHT]
 
asdfghj is offline   Trả Lời Với Trích Dẫn
Trả lời Gởi Ðề Tài Mới

Bookmarks


Quuyền Hạn Của Bạn
You may not post new threads
You may not post replies
You may not post attachments
You may not edit your posts

BB code is Mở
Smilies đang Mở
[IMG] đang Mở
HTML đang Tắt

Chuyển đến


Múi giờ GMT. Hiện tại là 01:47 PM.


Powered by: vBulletin Copyright ©2000-2024, Jelsoft Enterprises Ltd.
Inactive Reminders By mathscope.org
[page compression: 118.75 k/135.40 k (12.30%)]